NURS 332 test 3 - prep u

¡Supera tus tareas y exámenes ahora con Quizwiz!

Bell's palsy is a disorder of which cranial nerve? a) Vagus (X) b) Trigeminal (V) c) Vestibulocochlear (VIII) d) Facial (VII)

Bell's palsy is characterized by facial dysfunction, weakness, and paralysis. Trigeminal neuralgia is a disorder of the trigeminal nerve and causes facial pain. Mnire's syndrome is a disorder of the vestibulocochlear nerve. Guillain-Barr syndrome is a disorder of the vagus nerve.

Which anticholinergic agent is used to control tremor and rigidity in Parkinson disease?

Benztropine Mesylate

At which of the following spinal cord injury levels does the patient have full head and neck control?

C5

A patient recently noted difficulty maintaining his balance and controlling fine movements. The nurse explains that the provider will order diagnostic studies for the part of his brain known as the:

Cerebellum.

A nurse is preparing a client for a lumbar puncture. The client has heard about post-lumbar puncture headaches and asks what causes them. The nurse tells the client that these headches are caused by which of the following?

Cerebral spinal fluid leakage at the puncture site

The nurse is assisting the physician in completing a lumbar puncture. Which would the nurse note as a concern?

Cerebrospinal fluid is cloudy in nature.

Which of the following is one of the earliest signs of increased ICP?

Decreased level of consciousness (LOC)

The nurse is completing an assessment on a client with myasthenia gravis. Which of the following historical recounting provides the most significant evidence regarding when the disorder began?

Drooping eyelids

The nurse is performing an assessment for a patient in the clinic with Parkinson's disease. The nurse determines that the patient's voice has changed since the last visit and is now more difficult to understand. How should the nurse document this finding?

Dysphonia

The nurse working on the neurological unit is caring for a client with a basilar skull fracture. During assessment, the nurse expects to observe Battle's sign, which is a sign of basilar skill fracture. Which of the following correctly decribes Battle's sign?

Ecchymosis over the mastoid

An acoustic neuroma is a benign tumor of which cranial nerve?

Eighth

Which term refers to a method of recording, in graphic form, the electrical activity of a muscle?

Electromyography

Which of the following statements reflects nursing management of the patient with expressive aphasia?

Encourage the patient to repeat sounds of the alphabet.

What safety actions does the nurse need to take for a client receiving oxygen therapy who is undergoing magnetic resonance imaging (MRI)?

Ensure that no client care equipment containing metal enters the room where the MRI is located.

The nurse is caring for a client with aphasia. Which strategy will the nurse use to facilitate communication with the client?

Establishing eye contact

A client with a traumatic brain injury has developed increased intracranial pressure resulting in dibetes insipidus. While assessing the client, the nurse expects which of the following findings?

Excessive urine output and decreased urine osmolality

A client has been diagnosed as having global aphasia. The nurse recognizes that the client will be unable to perform which action?

Form words that are understandable or comprehend spoken words

A client is admitted reporting low back pain. How will the nurse best determine if the pain is related to a herniated lumbar disc?

Have the client lie on the back and lift the leg, keeping it straight.

The nurse is caring for a client with Guillain-Barré syndrome. Which assessment finding would indicate the need for oral suctioning?

Increased pulse rate, adventitious breath sounds

What is the only known risk factor for brain tumors?

Ionizing radiation

The nurse is caring for a client admitted with a stroke. Imaging studies indicate an embolus partially obstructing the right carotid artery. What type of stroke does the nurse know this client has?

Ischemic

A client in the intensive care unit (ICU) has a traumatic brain injury. The nurse must implement interventions to help control intracranial pressure (ICP). Which of the following are appropriate interventions to help control ICP?

Keep the client's neck in a neutral position (no flexing).

A client is admitted with weakness, expressive aphasia, and right hemianopia. The brain MRI reveals an infarct. The nurse understands these symptoms to be suggestive of which of the following findings?

Left-sided cerebrovascular accident (CVA)

What does the nurse recognize as the earliest sign of serious impairment of brain circulation related to increasing ICP?

Lethargy and stupor

The nurse is caring for a client diagnosed with a hemorrhagic stroke. The nurse recognizes that which intervention is most important?

Maintaining a patent airway

A nurse working on a medical-surgical floor walks into a patient's room to find the patient with an altered level of consciousness (LOC). Which of the following actions would be the first priority?

Maintenance of a patent airway

A client is exhibiting signs of increasing intracranial pressure (ICP). Which intravenous solution (IV) would the nurse anticipate hanging?

Mannitol(Osmitrol)

To meet the sensory needs of a client with viral meningitis, which of the following should the nurse do?

Minimize exposure to bright lights and noise.

A nurse is assessing a patient's urinary output as an indicator of diabetes insipidus. The nurse knows that an hourly output of what volume over 2 hours may be a positive indicator?

More than 200 mL/h

Which of the following is the initial diagnostic in suspected stroke?

Noncontrast computed tomography (CT)

A patient is diagnosed with an aggressive, primary malignant brain tumor. The nurse is aware that the glioma:

Originated within the brain tissue

A client presents to the emergency department stating numbness and tingling occurring down the left leg into the left foot. When documenting the experience, which medical terminology would the nurse be most correct to report?

Paresthesia

A patient has difficulty interpreting his awareness of body position in space. Which lobe is most likely to be damaged?

Parietal

The nurse educator is teaching nursing students about various types of brain tumors. The instructor recognizes that teaching has been effective when students correctly identify a client whose lab work indicates excessively high levels of thyroid stimulating hormone would most likely be diagnosed with which type of tumor?

Pituitary adenoma

A client on your unit is scheduled to have intracranial surgery in the morning. Which nursing intervention helps to avoid intraoperative complications, reduce cerebral edema, and prevent postoperative vomiting?

Restrict fluids before surgery.

The nurse is caring for a patient having a hemorrhagic stroke. What position in the bed will the nurse maintain this patient?

Semi-fowlers

Which neurotransmitter demonstrates inhibitory action, helps control mood and sleep, and inhibits pain pathways?

Serotonin

While providing information to a community group, the nurse tells them the primary initial symptoms of a hemorrhagic stroke are:

Severe headache and early change in level of consciousness

A client has a herniated disk in the region of the third and fourth lumbar vertebrae. Which nursing assessment finding most supports this diagnosis?

Severe lower back pain

The nurse is caring for a client immediately following a spinal cord injury (SCI). Which is an acute complication of SCI?

Spinal shock

Which cerebral lobe contains the auditory receptive areas?

Temporal

Which client should the nurse assess for degenerative neurologic symptoms?

The client with Huntington disease.

A client diagnosed with Parkinson's disease has developed slurred speech and drooling. The nurse knows that these symptoms indicate which of the following?

The disease has entered the late stages.

The nurse is performing a neurological assessment of a client who has sustained damage to the frontal cortex. Which of the following deficits will the nurse look for during assessment?

The inability to tell how a mouse and a cat are alike

A patient has an S5 spinal fracture from a fall. What type of assistive device will this patient require?

The patient will be able to ambulate independently.

The nurse is seeing the mother of a client who states, "I'm so relieved because my son's doctor told me his brain tumor is benign." The nurse knows what is true about benign brain tumors?

They can affect vital functioning

A patient having an acute stroke with no other significant medical disorders has a blood glucose level of 420 mg/dL. What significance does the hyperglycemia have for this patient?

This is significant for poor neurologic outcomes.

A nurse is teaching a community class that those experiencing symptoms of ischemic stroke need to enter the medical system early. The primary reason for this is which of the following?

Thrombolytic therapy has a time window of only 3 hours.

A patient diagnosed with an ischemic stroke should be treated within the first 3 hours of symptom onset with which of the following?

Tissue plasminogen activator (tPA)

An osmotic diuretic, such as mannitol, is given to the patient with increased intracranial pressure (IICP) for which of the following reasons?

To dehydrate the brain and reduce cerebral edema

A female patient with meningitis has a history of seizures. Which of the following actions by the nurse is appropriate while the patient is actively seizing?

Turn the patient to the side

A client is scheduled for a laminectomy to repair a herniated intervertebral disk. When developing the postoperative care plan, the nurse should include which action?

Turning the client from side to side, using the logroll technique

Which of the following is the chief cause of intracerebral hemorrhage (ICH)?

Uncontrolled hypertension

A client is scheduled for standard EEG testing to evaluate a possible seizure disorder. Which nursing intervention should the nurse perform before the procedure?

Withhold anticonvulsant medications for 24 to 48 hours before the exam

The nurse is performing a neurologic assessment on a client diagnosed with a stroke and cannot elicit a gag reflex. This deficit is related to which of the following cranial nerves?

X

A client reports light-headedness, speech disturbance, and left-sided weakness that have lasted for several hours. In the examination, an abnormal sound is auscultated in an artery leading to the brain. What is the term for the auscultated discovery?

bruit

A nurse is reading a journal article about stroke and the underlying causes associated with this condition. The nurse demonstrates understanding of the information when identifying which subtype of stroke as being due to atrial fibrillation?

cardio embolic

A client is sitting in a chair and begins having a tonic-clonic seizure. The most appropriate nursing response is to:

carefully move the client to a flat surface and turn him on his side.

Lower motor neuron lesions cause

flaccid muscles.

A nurse is working on a surgical floor. The nurse must logroll a client following a:

laminectomy

Bone density testing in clients with post-polio syndrome has demonstrated

low bone mass and osteoporosis.

A client who has been severely beaten is admitted to the emergency department. The nurse suspects a basilar skull fracture after assessing:

raccoon's eyes and Battle sign.

The nurse has completed evaluating the client's cranial nerves. The nurse documents impairment of the right cervical nerves (CN IX and CN X). Based on these findings, the nurse should instruct the client to

refrain from eating or drinking for now.

Which is a modifiable risk factor for transient ischemic attacks and ischemic strokes?

smoking

Which interventions are appropriate for a patient with increased ICP? Select all that apply.

-Administering prescribed antipyretics -Maintaining aseptic technique with the intraventricular catheter -Frequent oral care

The nurse is called to attend to a patient having a seizure in the waiting area. What nursing care is provided for a patient who is experiencing a convulsive seizure? (Select all that apply.)

-Loosening constrictive clothing -Positioning the patient on his or her side with head flexed forward -Providing for privacy

A nurse knows that, for a patient with an ischemic stroke, tPA is contraindicated if the blood pressure reading is:

190 mm Hg/120 mm Hg

Which Glasgow Coma Scale score is indicative of a severe head injury?

7

A client in the surgical intensive care unit has skeletal tongs in place to stabilize a cervical fracture. Protocol dictates that pin care should be performed each shift. When providing pin care for the client, which finding should the nurse report to the physician?

A small amount of yellow drainage at the left pin insertion site

The nurse is assessing a client newly diagnosed with myasthenia gravis. Which of the following signs would the nurse most likely observe? a) Diplopia and ptosis b) Numbness c) Patchy blindness d) Loss of proprioception

A, The initial manifestation of myasthenia gravis involves the ocular muscles, such as diplopia and ptosis. The remaining choices relate to multiple sclerosis.

Which cranial nerve is tested by listening to a ticking watch?

Acoustic

Which term refers to the failure to recognize familiar objects perceived by the senses?

Agnosia

A client with a history of atrial fibrillation has experienced a TIA. In an effort to reduce the risk of cerebrovascular accident (CVA), the nurse anticipates the priority medical treatment to include which of the following?

Anticoagulant therapy

The nurse is aware that burr holes may be used in neurosurgical procedures. Which of the following is a reason why a neurosurgeon may choose to create a burr hole in a patient?

Aspiration of a brain abscess

Which phase of a migraine headache usually lasts less than an hour?

Aura

A client with Parkinson's disease asks the nurse what their treatment is supposed to do since the disease is progressive. What would be the nurse's best response?

"Treatment aims at keeping you independent as long as possible."

A client is ordered to undergo CT of the brain with IV contrast. Before the test, the nurse should complete which action first?

Assess the client for medication allergies.

Which interventions would be recommended for a client with dysphagia? Select all that apply.

Assist the client with meals. Test the gag reflex before offering food or fluids. Allow ample time to eat.

Which of the following, if left untreated, can lead to an ischemic stroke?

Atrial fibrillation

A client with Guillain-Barré syndrome has paralysis affecting the respiratory muscles and requires mechanical ventilation. When the client asks the nurse about the paralysis, how should the nurse respond? a) "You'll first regain use of your legs and then your arms." b) "You'll be permanently paralyzed; however, you won't have any sensory loss." c) "The paralysis caused by this disease is temporary." d) "It must be hard to accept the permanency of your paralysis."

C, The nurse should inform the client that the paralysis that accompanies Guillain-Barré syndrome is only temporary. Return of motor function begins proximally and extends distally in the legs.

The nurse is caring for a client who underwent surgery to remove a spinal cord tumor. When conducting the postoperative assessment, the nurse notes the presence of a bulge at the surgical site. The nurse suspects the client is experiencing what complication from the surgery?

Cerebrospinal fluid leakage

Which of the following is the earliest sign of increasing ICP?

Change in level of consciousness (LOC)

A client is admitted to an acute care facility after an episode of status epilepticus. After the client is stabilized, which factor is most beneficial in determining the potential cause of the episode?

Compliance with the prescribed medication regimen

Which of the following is considered a central nervous system (CNS) disorder? a) Myasthenia gravis b) Guillain-Barré c) Bell's palsy d) Multiple sclerosis

D, Multiple sclerosis is an immune-mediated, progressive demyelinating disease of the CNS. Guillain-Barré, myasthenia gravis, and Bell's palsy are peripheral nervous system disorders.

A nurse is working in the neurologic intensive care unit and admits from the emergency department a patient with an inoperable brain tumor. Upon entering the room, the nurse observes that the patient is positioned like the person in part B of the accompanying image. Which posturing is the patient exhibiting?

Decerebrate

Which term refers to the shifting of brain tissue from an area of high pressure to an area of low pressure?

Herniation

A patient with epilepsy is having a seizure. Which of the following should the nurse do after the seizure?

Keep the patient to one side.

A nurse is preparing to administer an antiseizure medication to a client. Which of the following is an appropriate antiseizure medication?

Lamictal

A nurse is assisting during a lumbar puncture. How should the nurse position the client for this procedure?

Lateral recumbent, with chin resting on flexed knees

A client with tetraplegia cannot do his own skin care. The nurse is teaching the caregiver about the importance of maintaining skin integrity. Which of the following will the nurse most encourage the caregiver to do?

Maintain a diet for the client that is high in protein, vitamins, and calories.

A patient is admitted to a specialty care unit with a diagnosis of an upper motor neuron lesion. The nurse assesses the patient and documents the presence of:

Muscle spasticity.

After a seizure, the nurse should place the patient in which of the following positions to prevent complications?

Side-lying, to facilitate drainage of oral secretions

Which of the following is an inaccurate manifestation of Cushing's triad?

Tachycardia

A patient is exhibiting classic signs of a hemorrhagic stroke. What complaint from the patient would be an indicator of this type of stroke?

severe headache

An emergency department nurse is awaiting the arrival of a client with signs of an ischemic stroke that began 1 hour ago, as reported by emergency medical personnel. The treatment window for thrombolytic therapy is which of the following?

three hours

When communicating with a client who has sensory (receptive) aphasia, the nurse should:

use short, simple sentences.

A client has undergone surgery for a spinal cord tumor that was located in the cervical area. The nurse would be especially alert for which of the following?

Respiratory dysfunction

A 154-pound woman has been prescribed tPA (0.9 mg/kg) for an ischemic stroke. The nurse knows to give how many mg initially?

6.3 mg

A patient is admitted to the hospital with an ICP reading of 20 mm Hg and a mean arterial pressure of 90 mm Hg. What would the nurse calculate the CPP to be?

70 mm Hg

Low levels of the neurotransmitter serotonin lead to which of the following disease processes? a) Depression b) Seizures c) Parkinson's disease d) Myasthenia gravis

A A decrease of serotonin leads to depression. A decrease in the amount of acetylcholine causes myasthenia gravis. Parkinson's disease is caused by a depletion of dopamine. Decreased levels of GABA may cause seizures.

Which lobe of the brain is responsible for concentration and abstract thought?

Frontal

A patient sustained a head injury during a fall and has changes in personality and affect. What part of the brain does the nurse recognize has been affected in this injury?

Frontal lobe

A 37-year-old mother of three has just been diagnosed with a grade I meningioma. As part of patient education, the nurse tells the patient that:

Growth is slow and symptoms are caused by compression rather than tissue invasion.

A nurse is caring for a client with lower back pain who is scheduled for myelography using metrizamide (a water-soluble contrast dye). After the test, the nurse should place the client in which position?

Head of the bed elevated 45 degrees

Which of the following symptoms are indicative of a rapidly expanding acute subdural hematoma? Select all that apply.

Hemiparesis Decreased reactivity of the pupils Bradycardia Coma

Which is the most common motor dysfunction seen in clients diagnosed with stroke?

Hemiplegia

The nursing instructor gives their students an assignment of making a plan of care for a client with Huntington's disease. What would be important for the students to include in the teaching portion of the care plan?

How to facilitate tasks such as using both hands to hold a drinking glass

Which of the following areas of the brain are responsible for temperature regulation?

Hypothalamus

A client diagnosed with migraine headaches asks the nurse what he can do to help control the headaches and minimize the number of attacks he is having. What instructions should the nurse give this client?

Identify and avoid factors that precipitate or intensify an attack.

The nurse teaches the client with which disorder that the disease is due to decreased levels of dopamine in the basal ganglia of the brain?

Parkinson disease

After a transsphenoidal adenohypophysectomy, a client is likely to undergo hormone replacement therapy. A transsphenoidal adenohypophysectomy is performed to treat which type of cancer?

Pituitary carcinoma

The nurse is caring for a client following an aneurysm coiling procedure. The nurse documents that the client is experiencing Korsakoff syndrome. Which set of symptoms characterizes Korsakoff syndrome?

Psychosis, disorientation, delirium, insomnia, and hallucinations

A patient has been diagnosed with damage to Broca's area of the left frontal lobe. To document the extent of damage, the nurse would assess the patient's:

Speech.

A client who has just been diagnosed with mixed muscular dystrophy asks the nurse about the usual course of this disease. How should the nurse respond?

"You may experience progressive deterioration in all voluntary muscles."

What part of the brain controls and coordinates muscle movement? a) Cerebellum b) Cerebrum c) Midbrain d) Brain stem

A, The cerebellum, which is located behind and below the cerebrum, controls and coordinates muscle movement. Options B, C ,and D are incorrect

When educating a patient about the use of antiseizure medication, what should the nurse inform the patient is a result of long-term use of the medication in women? a) Anemia b) Osteoporosis c) Obesity d) Osteoarthritis

B, Because of bone loss associated with the long-term use of antiseizure medications, patients receiving antiseizure agents should be assessed for low bone mass and osteoporosis. They should be instructed about strategies to reduce their risks of osteoporosis (AANN, 2009).

Which of the following is the earliest sign of increasing intracranial pressure (ICP)?

Change in level of consciousness (LOC)

Which insult or abnormality can cause an ischemic stroke?

Cocaine use

A 53-year-old man presents to the emergency department with a chief complaint of inability to form words, and numbness and weakness of the right arm and leg. Where would you locate the site of injury?

Left frontoparietal region

What clinical manifestations does the nurse recognize when a patient has had a right hemispheric stroke?

Left visual field deficit

The provider orders the Romberg test for a patient. The nurse tells the patient that the provider wants to evaluate his equilibrium by assessing which cranial nerve?

VIII

Which of the following diagnostic studies provides visualization of cerebral blood vessels?

Cerebral angiography

A client experiences loss of consciousness, tongue biting, and incontinence, along with tonic and clonic phases of seizure activity. The nurse should document this episode as which type of seizure?

Generalized

The most common cause of cholinergic crisis includes which of the following? a) Overmedication b) Infection c) Compliance with medication d) Undermedication

A, A cholinergic crisis, which is essentially a problem of overmedication, results in severe generalized muscle weakness, respiratory impairment, and excessive pulmonary secretion that may result in respiratory failure. Myasthenic crisis is a sudden, temporary exacerbation of MG symptoms. A common precipitating event for myasthenic crisis is infection. It can result from undermedication. (

The nurse is discussing spinal cord injury (SCI) at a health fair at a local high school. The nurse relays that the most common cause of SCI is

Motor vehicle crashes

A client whose physical findings suggest a hyperpituitary condition undergoes an extensive diagnostic workup. Test results reveal a pituitary tumor, which necessitates a transsphenoidal hypophysectomy. The evening before the surgery, the nurse reviews preoperative and postoperative instructions given to the client earlier. Which postoperative instruction should the nurse emphasize?

"You must avoid coughing, sneezing, and blowing your nose."

The nursing is assessing a client who has been diagnosed with a pituitary adenoma, but has not yet started treatment. The client reports having increased heart rate, hand tremors, difficulty sleeping, weight loss and hyperthermia. The nurse anticipates the client will require blood work to assess for overproduction of which hormone?

Thyroid-stimulating hormone

A client is sitting in a chair and begins having a tonic-clonic seizure. The most appropriate nursing response is to: a) carefully move the client to a flat surface and turn him on his side. b) place an oral airway in the client's mouth to maintain an open airway. c) hold the client's arm still to keep him from hitting anything. d) allow the client to remain in the chair but move all objects out of his way.

A

A nurse is working in the neurologic intensive care unit and admits from the emergency department a patient with a severe head injury. Upon entering the room, the nurse observes that the patient is positioned like part A of the accompanying image. Which posturing is the patient exhibiting?

Decorticate

A client undergoes cerebral angiography for evaluation after an intracranial computed tomography scan revealed a subarachnoid hemorrhage. Afterward, the nurse checks frequently for signs and symptoms of complications associated with this procedure. Which findings indicate spasm or occlusion of a cerebral vessel by a clot?

Hemiplegia, seizures, and decreased level of consciousness (LOC)

A patient comes to the emergency department with severe pain in the face that was stimulated by brushing the teeth. What cranial nerve does the nurse understand can cause this type of pain?

V

A client with a spinal cord injury and subsequent urine retention receives intermittent catheterization every 4 hours. The average catheterized urine volume has been 550 ml. The nurse should plan to:

increase the frequency of the catheterizations.

After a plane crash, a client is brought to the emergency department with severe burns and respiratory difficulty. The nurse helps to secure a patent airway and attends to the client's immediate needs, then prepares to perform an initial neurologic assessment. The nurse should perform an:

evaluation of the corneal reflex response.

Ada Zontor, a 60-year-old bookkeeper, is a client with the neurological group where you practice nursing. Mrs. Zontor has been exhibiting neurological symptoms for several weeks and the neurologist is admitting her to hospital for extensive testing. Since diagnostics have not yet revealed the cause of her difficulties, which of her following comments would indicate the need for further client education? a) There are several types of tests to see what's causing the tingling in my fingers and toes. b) All of her comments indicate need for further client education. c) It's good to know the continual tingling in my fingers and toes is not connected with my nervous system! d) I need to be careful with my allergy to seafood!

C, The nervous system consists of the brain, spinal cord, and peripheral nerves.

A client with cerebral metastasis suddenly experiences a seizure for which phenytoin 10 mg/kg intravenously is ordered as an initial loading dose. The client weighs 165 pounds. How many milligrams of phenytoin should the client receive? Enter the number ONLY.

750

A client has been hospitalized for diagnostic testing. The client has just been diagnosed with multiple sclerosis, which the physician explains is an autoimmune disorder. How would the nurse explain an autoimmune disease to the client? a) A disorder in which killer T cells and autoantibodies attack or destroy natural cells—those cells that are "self" b) A disorder in which histocompatible cells attack the immunoglobulins c) A disorder in which the body does not have enough immunoglobulins d) A disorder in which the body has too many immunoglobulins

A, Autoimmune disorders are those in which killer T cells and autoantibodies attack or destroy natural cells—those cells that are "self." Autoantibodies, antibodies against self-antigens, are immunoglobulins. They target histocompatible cells, cells whose antigens match the person's own genetic code. Autoimmune disorders are not caused by too many or too few immunoglobulins, and histocompatible cells do not attack immunoglobulins in an autoimmune disorder.

A patient has been diagnosed with meningococcal meningitis at a community living home. When should prophylactic therapy begin for those who have had close contact with the patient? a) Within 72 hours after exposure b) Within 24 hours after exposure c) Within 48 hours after exposure d) Therapy is not necessary prophylactically and should only be used if the person develops symptoms.

B, People in close contact with patients with meningococcal meningitis should be treated with antimicrobial chemoprophylaxis using rifampin (Rifadin), ciprofloxacin hydrochloride (Cipro), or ceftriaxone sodium (Rocephin). Therapy should be started within 24 hours after exposure because a delay in the initiation of therapy limits the effectiveness of the prophylaxis

A client, suspected of having a distortion of cerebral arteries and veins, is scheduled for a cerebral angiography. What would the nurse tell the client about the upcoming test? a) That sedatives, coffee, tea, and soft drinks that contain caffeine will be withheld for at least 8 hours before the test to avoid affecting the diagnostic findings. b) The client will have to stay in a dark quiet room. c) Contrast will be given and a rapid sequence of radiographs will be taken. d) The client will have to shampoo his or her hair.

C, A radiopaque dye is injected into the right or left carotid artery, the brachial artery, or the femoral artery. A rapid sequence of radiographs is taken as the dye circulates through the cerebral arteries and veins. For cerebral angiography options A, B, and D do not apply.

A patient is admitted to the hospital with an ICP reading of 20 mm Hg and a mean arterial pressure of 90 mm Hg. What would the nurse calculate the CPP to be? a) 50 mm Hg b) 60 mm Hg c) 70 mm Hg d) 80 mm Hg

C, Changes in ICP are closely linked with cerebral perfusion pressure (CPP). The CPP is calculated by subtracting the ICP from the mean arterial pressure (MAP). For example, if the MAP is 100 mm Hg and the ICP is 15 mm Hg, then the CPP is 85 mm Hg. The normal CPP is 70 to 100 mm Hg (Hickey, 2009)

When obtaining the vital signs of a client with multiple traumatic injuries, a nurse detects bradycardia, bradypnea, and systolic hypertension. The nurse must notify the physician immediately because these findings may reflect which complication?

Increased intracranial pressure (ICP)

A client with weakness and tingling in both legs is admitted to the medical-surgical unit with a tentative diagnosis of Guillain-Barré syndrome. On admission, which assessment is most important for this client?

Lung auscultation and measurement of vital capacity and tidal volume

An emergency department nurse is interviewing a client with signs of an ischemic stroke that began 2 hours ago. The client reports that she had a cholecystectomy 6 weeks ago and is taking digoxin, coumadin, and labetelol. This client is not eligible for thrombolytic therapy for which of the following reasons?

she is taking coumadin

A client recently experienced a stroke with accompanying left-sided paralysis. His family voices concerns about how to best interact with him. They report the client doesn't seem aware of their presence when they approach him on his left side. What advice should the nurse give the family?

"The client is unaware of his left side. You should approach him on the right side."

Thrombolytic therapy should be initiated within what time frame of an ischemic stroke to achieve the best functional outcome?

3 hours

A nurse is assessing a patient's urinary output as an indicator of diabetes insipidus. The nurse knows that an hourly output of what volume over 2 hours may be a positive indicator? a) More than 200 mL/h b) 50 to 100 mL/h c) 150 to 200 mL/h d) 100 to 150 mL/h

A, For patients undergoing dehydrating procedures, vital signs, including blood pressure, must be monitored to assess fluid volume status. An indwelling urinary catheter is inserted to permit assessment of renal function and fluid status. During the acute phase, urine output is monitored hourly. An output greater than 200 mL per hour for 2 consecutive hours may indicate the onset of diabetes insipidus (Hickey, 2009).

A client with myasthenia gravis is admitted with an exacerbation. The nurse is educating the client about plasmapherisis and explains this in which of the following statements? a) Antibodies are removed from the plasma. b) Mestinon therapy is initiated. c) Immune globulin is given intravenously. d) The thymus gland is removed.

A, Plasmapheresis is a technique in which antibodies are removed from plasma and the plasma is returned to the client. The other three choices are appropriate treatments for myasthenia gravis, but are not related to plasmapheresis

An unresponsive patient is brought to the ED by a family member. The family states, "We don't know what happened." Which of the following is the priority nursing intervention?

Assess for a patent airway.

A patient is admitted to the emergency room with a fractured skull sustained in a motorcycle accident. The nurse notes fluid leaking from the patient's ears. The nurse knows this is a probable sign of which type of skull fracture?

Basilar

To meet the sensory needs of a client with viral meningitis, which of the following should the nurse do? Choose the correct option. a) Avoid physical contact with family members b) Promote an active range of motion c) Minimize exposure to bright lights and noise d) Increase environmental stimuli

C, Photophobia and hypersensitivity to environmental stimuli are the common clinical manifestations of meningeal irritation and infection. Therefore, the nurse should provide a calm environment having less stressful stimuli to such clients.

Upper motor neuron lesions cause a) flaccid paralysis. b) absent or decreased reflexes. c) no muscle atrophy. d) decreased muscle tone.

C,Upper motor neuron lesions do not cause muscle atrophy but do cause loss of voluntary control. Lower motor neuron lesions cause decreased muscle tone. Lower motor neuron lesions cause flaccid paralysis. Lower motor neuron lesions cause absent or decreased reflexes

Which of the following outcomes would be most appropriate to include in the plan of care for a client diagnosed with a muscular dystrophy?

Client participates in activities of daily living using adaptive devices.

The nurse is offering suggestions regarding reproductive options to a husband and paraplegic wife. Which option is most helpful?

Conception is not impaired; the birth process is determined with the physician.

When the nurse observes that the patient has extension and external rotation of the arms and wrists, and extension, plantar flexion, and internal rotation of the feet, she records the patient's posturing as which of the following?

Decerebrate

The greatest risk of seizures for clients with brain tumors occurs in those who have tumors in which regions of the brain? Select all that apply.

Frontal Parietal Temporal

Cranial nerve IX is also known as which of the following?

Glossopharyngeal

The nurse working on a neurological unit is mentoring a nursing student. The student asks about a client who has sustained a primary and and secondary brain injury. The nurse correctly tells the student which of the following, related to the primary injury?

It results from initial damage to the brain from the traumatic event.

Which of the following is a late symptom of spinal cord compression?

Paralysis

The pre-nursing class is learning about the nervous system in their anatomy class. What part of the nervous system would the students learn is responsible for digesting food and eliminating body waste?

Parasympathetic

A 55-year-old female client presents at the walk-in clinic complaining of feeling like a mask is on her face. While doing the initial assessment, the nurse notes the demonstration of a pill-rolling movement in the right hand and a stooped posture. Physical examination shows bradykinesia and a shuffling gait. What would the nurse suspect is the causative factor for these symptoms?

Parkinson's disease

The nurse is caring for a client with dysphagia. Which intervention would be contraindicated while caring for this client?

Placing food on the affected side of the mouth

Which statement indicates appropriate nursing intervention for a client with post-polio syndrome?

Provide care aimed at slowing the loss of strength and maintaining overall well-being.

A nurse is planning care for a client who experienced a stroke in the right hemisphere of his brain. What should the nurse do?

Provide close supervision because of the client's impulsiveness and poor judgment.

The nurse practitioner is able to correlate a patient's neurologic deficits with the location in the brain affected by ischemia or hemorrhage. For a patient with a left hemispheric stroke, the nurse would expect to see:

Right-sided paralysis.

A female patient is receiving hypothermic treatment for uncontrolled fevers related to increased intracranial pressure (ICP). Which of the following assessment finding requires immediate intervention?

Shivering

Which of the following is the most common side effect of tissue plasminogen activator (tPA)?

bleeding

A nurse is completing a neurological assessment and determines that the client has significant visual deficits. Considering the functions of the lobes of the brain, which area will most likely contain the neurologic deficit?

occipital

Which of the following is one of the earliest signs of increased ICP? a) Lethargy b) Decreased level of consciousness (LOC) c) Headache d) Coma

B, Decreasing LOC is one of the earliest signs of increased ICP.

A client is diagnosed with a brain tumor. The nurse's assessment reveals that the client has difficulty interpreting visual stimuli. Based on these findings, the nurse suspects injury to which lobe of the brain?

Occipital

A client was hit in the head with a ball and knocked unconscious. Upon arrival at the emergency department and subsequent diagnostic tests, it was determined that the client suffered a subdural hematoma. The client is becoming increasingly symptomatic. How would the nurse expect this subdural hematoma to be classified?

acute

A nurse is working on a neurological unit with a nursing student who asks the difference between primary and secondary headaches. The nurse's correct response will include which of the following statements?

"A secondary headache is associated with an organic cause, such as a brain tumor."

The spouse of a client with terminal brain cancer asks the nurse about hospice. Which statement by the nurse best describes hospice care?

"Clients and families are the focus of hospice care."

To evaluate a client's cerebellar function, a nurse should ask:

"Do you have any problems with balance?"

A patient arrives to have an MRI done in the outpatient department. What information provided by the patient warrants further assessment to prevent complications related to the MRI?

"I am trying to quit smoking and have a patch on."

The nurse is preparing to administer tissue plasminogen activator (t-PA) to a patient who weighs 132 lb. The order reads 0.9 mg/kg t-PA. The nurse understands that 10% of the calculated dose is administered as an IV bolus over 1 minute, and the remaining dose (90%) is administered IV over 1 hour via an infusion pump. How many milligrams IV bolus over 1 minute will the nurse initially administer?

5.4

A patient has difficulty interpreting his awareness of body position in space. Which lobe is most likely to be damaged? a) Parietal b) Occipital c) Temporal d) Frontal

A, The parietal lobe is the primary sensory cortex. It is essential to a person's awareness of his body in space, as well as orientation in space and spatial relations.

A client with quadriplegia is in spinal shock. What finding should the nurse expect?

Absence of reflexes along with flaccid extremities

A patient sustained a head trauma in a diving accident and has a cerebral hemorrhage located within the brain. What type of hematoma is this classified as?

An intracerebral hematoma

A nurse is caring for a client who has returned to his room after a carotid endarterectomy. Which action should the nurse take first?

Ask the client if he has trouble breathing.

During recovery from a stroke, a client is given nothing by mouth to help prevent aspiration. To determine when the client is ready for a liquid diet, the nurse assesses the client's swallowing ability once per shift. This assessment evaluates: a) cranial nerves I and II. b) cranial nerves IX and X. c) cranial nerves III and V. d) cranial nerves VI and VIII.

B, Swallowing is a motor function of cranial nerves IX and X. Cranial nerves I, II, and VIII don't possess motor functions. The motor functions of cranial nerve III include extraocular eye movement, eyelid elevation, and pupil constriction. The motor function of cranial nerve V is chewing. Cranial nerve VI controls lateral eye movement.

A nurse assesses the patient's LOC using the Glasgow Coma Scale. What score indicates severe impairment of neurologic function? a) 9 b) 3 c) 12 d) 6

B, LOC, a sensitive indicator of neurologic function, is assessed based on the criteria in the Glasgow Coma Scale: eye opening, verbal response, and motor response (Barlow, 2012). The patient's responses are rated on a scale from 3 to 15. A score of 3 indicates severe impairment of neurologic function, brain death, or pharmacologic inhibition of the neurologic response. A score of 15 indicates that the patient is fully responsive (see Chapter 68)

The Family Nurse Practitioner is performing the physical examination of a client with a suspected neurologic disorder. In addition to assessing other parts of the body, the nurse should assess for neck rigidity. Which method should help the nurse assess for neck rigidity correctly? a) Gently pressing the bones on the neck b) Moving the head and chin toward the chest c) Moving the head toward both sides d) Lightly tapping the lower portion of the neck to detect sensation

B, The neck is examined for stiffness or abnormal position. The presence of rigidity is assessed by moving the head and chin toward the chest. The nurse should not maneuver the neck if a head or neck injury is suspected or known. The neck should also not be maneuvered if trauma to any part of the body is evident. Moving the head toward the sides or pressing the bones on the neck will not help assess for neck rigidity correctly. While assessing for neck rigidity, sensation at the neck area is not to be assessed

When caring for a client who is post-intracranial surgery what is the most important parameter to monitor?

Body temperature

In the divisions of the nervous systems, the basic structure is the neuron. The function of the neuron is determined by the direction of impulse transmission. Which part of the neuron is responsible for conducting impulses away from the cell body? a) Nucleus b) Dendrite c) Efferent nerve fibers d) Afferent nerve fibers

C, An axon is a nerve fiber that projects and conducts impulses away from the cell body. It is therefore called an efferent ("away from") nerve fiber

While making your initial rounds after coming on shift, you find a client thrashing about in bed complaining of a severe headache. The client tells you the pain is behind his right eye which is red and tearing. What type of headache would you suspect this client of having?

Cluster

The nurse reviews the patient's drug regimen for treatment of a brain tumor. She explains to the patient why one of the following drugs would not be prescribed, even though it might have therapeutic benefits. Which drug would not be prescribed for this patient?

Coumadin

The nurse obtains a Snellen eye chart when assessing cranial nerve function. Which cranial nerve is the nurse testing when using the chart? a) CN I b) CN IV c) CN III d) CN II

D, The nurse assesses vision and thus the optic nerve (cranial nerve II) by use of a Snellen eye chart.

Which of the following assessment findings would indicate an increasing intracranial pressure (ICP) in a client with head trauma? Select all that apply.

Elevated systolic blood pressure Wide pulse pressure

After a stroke, a client is admitted to the facility. The client has left-sided weakness and an absent gag reflex. He's incontinent and has a tarry stool. His blood pressure is 90/50 mm Hg, and his hemoglobin is 10 g. Which nursing intervention is a priority for this client?

Elevating the head of the bed to 30 degrees

The nurse is caring for a client with mid-to-late stage of an inoperable brain tumor. What teaching is important for the nurse to do with this client?

Explaining hospice care and services

The nurse is caring for a client who is scheduled for surgery to relieve pressure on a compressed nerve. The compression does not involve the spinal cord. What kind of spinal nerve root compression does the nurse know this is?

Extramedullary

A patient has recently been diagnosed with an acoustic neuroma. The nurse helps the patient understand that:

Hearing loss usually occurs.

A client is diagnosed with a brain angioma. When teaching the client about the risks associated with this type of brain tumor, the nurse would educate about signs and symptoms associated with which condition?

Hemorrhagic stroke

The nurse is assigned to care for clients with SCI on a rehabilitation unit. Which signs does the nurse recognize as clinical manifestations of autonomic dysreflexia? Select all that apply.

Hypertension Diaphoresis Nasal congestion

A client arrives at the ED via ambulance following a motorcycle accident. The paramedics state the client was found unconscious at the scene but briefly regained consciousness during transport to the hospital. Upon initial assessment, the client's GCS score is 7. The nurse anticipates which action?

Immediate craniotomy

A diagnostic test has determined that the appropriate diet for the client with a left cerebrovascular accident (CVA) should include honey thickened liquids. Which of the following is the priority nursing diagnosis for this client?

Impaired swallowing

The nurse is caring for a patient postoperatively after intracranial surgery for the treatment of a subdural hematoma. The nurse observes an increase in the patient's blood pressure from the baseline and a decrease in the heart rate from 86 to 54. The patient has crackles in the bases of the lungs. What does the nurse suspect is occurring?

Increased ICP

A nurse assesses a patient who has been diagnosed with having a pituitary adenoma that is pressing on the third ventricle. The nurse looks for the associated sign/symptom. What is that sign/symptom?

Increased intracranial pressure

Which nursing diagnosis takes the highest priority for a client with parkinsonian crisis?

Ineffective airway clearance

A nurse working on a medical-surgical floor walks into a patient's room to find the patient with an altered level of consciousness (LOC). Which of the following nursing diagnoses would be the first priority for the plan of care?

Ineffective airway clearance related to altered LOC

A nurse practitioner provides health teaching to a patient who has difficulty managing hypertension. This patient is at an increased risk of which type of stroke?

Intracerebral hemorrhage

A nurse is working with a student nurse who is caring for a client with an acute bleeding cerebral aneurysm. Which action by the student nurse requires further intervention?

Keeping the client in one position to decrease bleeding

The most important nursing priority of treatment for a patient with an altered LOC is to:

Maintain a clear airway to ensure adequate ventilation.

A client who complains of recurring headaches, accompanied by increased irritability, photophobia, and fatigue is asked to track the headache symptoms and occurrence on a calendar log. Which is the best nursing rationale for this action?

Migraines often coincide with menstrual cycle.

A nurse is continually monitoring a client with a traumatic brain injury for signs of increasing intracranial pressure. The cranial vault contains brain tissue, blood, and cerebrospinal fluid; an increase in any of the components causes a change in the volume of the others. This hypothesis is called which of the following?

Monro-Kellie

A patient 3 days postoperative from a craniotomy informs the nurse, "I feel something trickling down the back of my throat and I taste something salty." What priority intervention does the nurse initiate?

Notify the physician of a possible cerebrospinal fluid leak.

The nurse is caring for a comatose client. The nurse knows she should assess the client's motor response. Which method may the nurse use to assess the motor response?

Observing the client's response to painful stimulus

A nurse is completing a neurological assessment and determines that the client has significant visual deficits. A brain tumor is considered. Considering the functions of the lobes of the brain, which area will most likely contain the neurologic deficit?

Occipital

A patient with neurological infection develops cerebral edema from syndrome of inappropriate antidiuretic hormone (SIADH). Which of the following is an important nursing action for this patient?

Restricting fluid intake and hydration

Following a generalized seizure in a client, which nursing assessment is a priority for detailing the event?

Seizure was 1 minute in duration including tonic-clonic activity.

Which is indicative of a right hemisphere stroke?

Spatial-perceptual deficits

Which of the following types of hematoma results from venous bleeding with blood gradually accumulating in the space below the dura?

Subdural

Which condition occurs when blood collects between the dura mater and arachnoid membrane?

Subdural hematoma

A client diagnosed with Huntington's disease has developed severe depression. What would be most important for the nurse to assess for?

Suicidal ideations

Autonomic dysreflexia can occur with spinal cord injuries above which of the following levels?

T6

Which of the following is not a manifestation of Cushing's Triad?

Tachycardia

A client has sustained a traumatic brain injury with involvement of the hypothalamus. The nurse is concerned about the development of diabetes insipidus. Which of the following would be an appropriate nursing intervention to monitor for early signs of diabetes insipidus?

Take daily weights.

The nurse is caring for a client with traumatic brain injury (TBI). Which clinical finding, observed during the reassessment of the client, causes the nurse the most concern?

Temperature increase from 98.0°F to 99.6°F

The nurse received the report from a previous shift. One of her clients was reported to have a history of basilar skull fracture with otorrhea. What assessment finding does the nurse anticipate?

The client has cerebral spinal fluid (CSF) leaking from the ear.

The nurse is seeing a client who is suspected of having a glioblastoma multiforme tumor. The nurse anticipates the client will require which diagnostic test to confirm the client has this form of brain tumor?

Tissue biopsy

The spinal cord is composed of 31 pairs of spinal nerves. How many pairs of thoracic nerves are contained within the spinal column?

Twelve

Which of the following teaching points is a priority in the management of symptoms for a client with Bell's palsy?

Use ophthalmic lubricant and protect the eye.

If warfarin is contraindicated as a treatment for stroke, which medication is the best option?

aspirin

A client experienced a stroke that damaged the hypothalamus. The nurse should anticipate that the client will have problems with:

body temperature control.

While snowboarding, a fell and sustained a blow to the head, resulting in a loss of consciousness. The client regained consciousness within an hour after arrival at the ED, was admitted for 24-hour observation, and was discharged without neurologic impairment. What would the nurse expect this client's diagnosis to be?

concussion

A client is waiting in a triage area to learn the medical status of family members following a motor vehicle accident. The client is pacing, taking deep breaths, and handwringing. Considering the effects in the body systems, the nurse anticipates that the liver will:

convert glycogen to glucose for immediate use.

During recovery from a stroke, a client is given nothing by mouth to help prevent aspiration. To determine when the client is ready for a liquid diet, the nurse assesses the client's swallowing ability once per shift. This assessment evaluates:

cranial nerves IX and X.

The initial sign of increasing ICP includes

decreased level of consciousness.

If a client has a lower motor neuron lesion, the nurse would expect the client to exhibiT?

decreased muscle tone.

A nurse is monitoring a client for increasing intracranial pressure (ICP). Early signs of increased ICP include:

diminished responsiveness.

A client is suspected of having amyotrophic lateral sclerosis (ALS). To help confirm this disorder, the nurse prepares the client for various diagnostic tests. The nurse expects the physician to order:

electromyography (EMG).

A nurse assesses the patient's LOC using the Glasgow Coma Scale. What score indicates severe impairment of neurologic function?

3

A potential complication of a hemorrhagic stroke is interference with the ability of the arachnoid villi to absorb CSF. Therefore, fluid in the ventricles increase beyond the amount that is usually absorbed daily, which is:

350 to 375 mL of this is usually absorbed daily.

A nurse is providing education about migraine headaches to a community group. The cause of migraines has not been clearly demonstrated, but is related to vascular disturbances. A member of the group asks about familial tendencies. The nurse's correct reply will be which of the following?

"There is a strong familial tendency."

The client with Guillain-Barré syndrome is scheduled for plasmapheresis and is questioning how this process works. Which of the following statements by the nurse best describes plasmapheresis in the management of this syndrome?

'Antibodies that triggered the autoimmune response are removed from your blood.'

The causes of acquired seizures include what? (Mark all that apply.)

-Cerebrovascular disease -Metabolic and toxic conditions -Brain tumor -Drug and alcohol withdrawal

The nurse is providing discharge teaching for a client who was admitted to hospital after having complex partial seizures secondary to a glioma. The client has been prescribed levetiracetam to manage the seizures. What should the nurse include in the discharge teaching for this medication?

"Driving a car should be avoided until the you know how this medication effects you."

A nurse is instructing the spouse of a client who suffered a stroke about the use of eating devices the client will be using. During the teaching, the spouse starts to cry and states, "One minute he is laughing, and the next he's crying; I just don't understand what's wrong with him." Which statement is the best response by the nurse?

"Emotional lability is common after a stroke, and it usually improves with time."

The nurse is seeing a client who has just been diagnosed with a meningioma. The client states he is confused because the provider stated, "If you have to be diagnosed with a brain tumor, this is the least harmful." The client asks the nurse for clarification. How should the nurse respond?

"I am unable to interpret what your provider meant by making that statement; however, it is true that meningiomas are slow growing tumors that are not typically fatal."

A client with post-polio syndrome displays fatigue and decreased muscle strength. How should the nurse best respond to the client?

"Intravenous immunoglobulin infusion may help you."

A client who recently experienced a stroke tells the nurse that he has double vision. Which nursing intervention is the most appropriate? a) Alternatively patch one eye every 2 hours. b) Encourage the client to close his eyes. c) Instill artificial tears. d) Turn out the lights in the room.

A,Patching one eye at a time relieves diplopia (double vision). Closing the eyes and making the room dark aren't the most appropriate options because they deprive the client of sensory input. Artificial tears relieve eye dryness but don't treat diplopia

Which term refers to the inability to coordinate muscle movements, resulting in difficulty walking?

Ataxia

An older client complains of a constant headache. A physical examination shows papilledema. What may the symptoms indicate in this client?

Brain tumor

The provider diagnoses the patient as having had an ischemic stroke. The etiology of an ischemic stroke would include which of the following?

Cardiogenic emboli

A patient has a deficiency of the neurotransmitter serotonin. The nurse is aware that this deficiency can lead to:

Depression.

Which is a sympathetic effect of the nervous system?

Dilated pupils

The statements presented here match nursing interventions with nursing diagnoses. Which statements are true for a client with a stroke? Select all that apply.

Impaired swallowing: Provide a pureed diet. Disturbed sensory perception: Stand on the client's unaffected side. Impaired verbal communication: Repeat words and instructions.

A patient comes to the emergency department with a large scalp laceration after being struck in the head with a glass bottle. After assessment of the patient, what does the nurse do before the physician sutures the wound?

Irrigates the wound to remove debris

A client who has sustained a head injury to the parietal lobe cannot identify a familiar object by touch. The nurse knows that this deficit is which of the following?

Tactile agnosia

A client is diagnosed with a tumor of the temporal lobe. When developing the client's plan of care the nurse would plan interventions to address problems with which areas of functioning? Select all that apply.

Understanding language Emotions Memory

In your assessment of a 39-year-old victim of a motor vehicle collision, he directly and accurately answers your questions. Beginning at his head, you note a contusion to his forehead; the client reports a headache. As you assess his pupils, what reaction would confirm your suspicion of increasing intracranial pressure?

Unequal response

Which of the following are used to help reduce ICP?

Using a cervical collar

The nurse who is employed in a neurologist's office is performing a history and assessment on a client experiencing hearing difficulty. The nurse is most correct to gather equipment to assess the function of cranial nerve:

VIII

The client presents to the walk-in clinic with fever, nuchal rigidity, and headache. Which of the following assessment findings would be most significant in the diagnosis of this client?

Vector bites

The nurse is providing information about strokes to a community group. Which of the following would the nurse identify as the primary initial symptoms of an ischemic stroke?

Weakness on one side of the body and difficulty with speech

A nurse is caring for a client with an injury to the central nervous system. When caring for a client with a spinal cord insult slowing transmission of the motor neurons, the nurse would anticipate a delayed reaction in:

response due to interrupted impulses from the central nervous system

Which of the following neurotransmitters are deficient in myasthenia gravis?

Acetylcholine

A stroke victim is experiencing memory loss and impaired learning capacity. The nurse knows that brain damage has most likely occurred in which lobe?

frontal

Which of the following is a chronic, degenerative, progressive disease of the central nervous system characterized by the occurrence of small patches of demyelination in the brain and spinal cord? a) Multiple sclerosis b) Creutzfeldt-Jakob disease c) Huntington disease d) Parkinson's disease

A, The cause of MS is not known and the disease affects twice as many women as men. Parkinson's disease is associated with decreased levels of dopamine caused by destruction of pigmented neuronal cells in the substantia nigra in the basal ganglia of the brain. Huntington disease is a chronic, progressive, hereditary disease of the nervous system that results in progressive involuntary dancelike movements and dementia. Creutzfeldt-Jakob disease is a rare, transmissible, progressive fatal disease of the central nervous system characterized by spongiform degeneration of the gray matter of the brain.

The nurse is completing a neurological assessment and uses the whisper test to assess which cranial nerve?

Acoustic

Which of the following tests confirms the diagnosis of myasthenia gravis (MG)? a) Computed tomography (CT) scan b) Tensilon test c) Electromyogram (EMG) d) Serum studies

B, Edrophonium chloride (Tensilon) is an acetylcholinesterase inhibitor that stops the breakdown of acetylcholine. The drug is used because it has a rapid onset of 30 seconds and a short duration of 5 minutes. Immediate improvement in muscle strength after administration of this agent represents a positive test and usually confirms the diagnosis. The presence of acetylcholine receptor antibodies is identified in serum. Repetitive nerve stimulation demonstrates a decrease in successive action potentials. The thymus gland may be enlarged in MG, and a T scan of the mediastinum is performed to detect thymoma or hyperplasia of the thymus.

What drug, prescribed for Parkinson's disease, has neuroprotective properties? a) Amantadine (Symmetrel) b) Selegiline (Eldepryl) c) Levodopa (Larodopa) d) Bromocriptine (Parlodel)

B, Selegiline (Eldepryl), has neuroprotective properties; dopaminergics such as levodopa (Larodopa) or levodopa-carbidopa (Sinemet); amantadine (Symmetrel); dopamine agonists such as bromocriptine (Parlodel); apomorphine (Apokyn), the newest approved drug; and anticholinergics such as benztropine (Cogentin) are prescribed.

A high school soccer player sustained five concussions before she was told that she should never play contact sports again. After her last injury, she began experiencing episodes of double vision. She was told that she had most likely incurred damage to which cranial nerve? a) V (Trigeminal) b) VI (Abducens) c) IV (Trochlear) d) VII (Facial)

B, The abducens cranial nerve supports movement of the eye laterally. Damage to the nerve can cause double vision.

A client is admitted to an acute care facility after an episode of status epilepticus. After the client is stabilized, which factor is most beneficial in determining the potential cause of the episode? a) The type of anticonvulsant prescribed to manage the epileptic condition b) Compliance with the prescribed medication regimen c) Recent weight gain and loss d) Recent stress level

B, The most common cause of status epilepticus is sudden withdraw of anticonvulsant therapy. The type of medication prescribed, the client's stress level, and weight change don't contribute to this condition.

A nurse is caring for a patient who is exhibiting signs and symptoms of autonomic dysreflexia. What clinical manifestations would the nurse expect in this patient?

Bradycardia and hypertension

The nurse is performing an assessment of cranial nerve function and asks the patient to cover one nostril at a time to see if the patient can smell coffee, alcohol, and mint. The patient is unable to smell any of the odors. The nurse is aware that the patient has a dysfunction of which cranial nerve? a) CN III b) CN II c) CN I d) CN IV

C, Cranial nerve (CN) I is the olfactory nerve, which allows the sense of smell. Testing of CN I is done by having the patient identify familiar odors with eyes closed, testing each nostril separately. An inability to smell an odor is a significant finding, indicating dysfunction of this nerve.

Which lobe of the brain is responsible for concentration and abstract thought? a) Occipital b) Parietal c) Frontal d) Temporal

C, The major functions of the frontal lobe are concentration, abstract thought, information storage or memory, and motor function. The parietal lobe analyzes sensory information such as pressure, vibration, pain, and temperature. The occipital lobe is the primary visual cortex. The temporal lobe contains the auditory receptive areas located around the temples.

When providing discharge teaching for a client with multiple sclerosis (MS), the nurse should include which instruction? a) "Limit your fruit and vegetable intake." b) "Avoid taking daytime naps." c) "Avoid hot baths and showers." d) "Restrict fluid intake to 1,500 ml/day."

C, The nurse should instruct a client with MS to avoid hot baths and showers because they may exacerbate the disease. The nurse should encourage daytime naps because fatigue is a common symptom of MS. A client with MS doesn't require food or fluid restrictions.

The nurse is caring for a patient postoperatively after intracranial surgery for the treatment of a subdural hematoma. The nurse observes an increase in the patient's blood pressure from the baseline and a decrease in the heart rate from 86 to 54. The patient has crackles in the bases of the lungs. What does the nurse suspect is occurring? a) Infection b) Increase in cerebral perfusion pressure c) Increased ICP d) Exacerbation of uncontrolled hypertension

C,Increased ICP and bleeding are life threatening to the patient who has undergone intracranial surgery. An increase in blood pressure and decrease in pulse with respiratory failure may indicate increased ICP.

Which type of brain injury has occurred if the client can be aroused with effort but soon slips back into unconsciousness?

Contusion

The critical care nurse is giving report on a client they are caring for. The nurse uses the Glasgow Coma Scale (GCS) to assess the level of consciousness (LOC) of a female client and reports to the on-coming nurse that the client has an LOC of 6. What does an LOC score of 6 in a client indicate? a) Stupor b) Somnolence c) Normal d) Comatose

D, The GSC is used to measure the LOC. The scale consists of three parts: eye opening response, best verbal response, and best motor response. A normal response is 15. A score of 7 or less is considered comatose. Therefore, a score of 6 indicates the client is in a state of coma, and not in any other state such as stupor or somnolence. The evaluations are recorded on a graphic sheet where connecting lines show an increase or decrease in the LOC

The nurse in the neurologic ICU is caring for a client who sustained a severe brain injury. Which nursing measures will the nurse implement to help control intracranial pressure (ICP)?

Maintain cerebral perfusion pressure from 50 to 70 mm Hg

During assessment of cognitive impairment, post-stroke, the nurse documents that the patient was experiencing memory loss and impaired learning capacity. The nurse knows that brain damage has most likely occurred in which lobe?

frontal

The nurse is caring for a client in the emergency department with a diagnosis of head trauma secondary to a motorcycle accident. The nurse aide is assigned to clean the client's face and torso. The nurse would provide further instruction after seeing that the nurse aide:

moved the client's head to clean behind the ears.

The school nurse notes a 6-year-old running across the playground with friends. The child stops in mid-stride, freezing for a few seconds. Then the child resumes their progress across the playground. The school nurse suspects what in this child?

An absence seizure

A client with Parkinson's disease has been receiving levodopa as treatment for the past 7 years. The client comes to the facility for an evaluation and the nurse observes facial grimacing, head bobbing, and smacking movements. The nurse interprets these findings as which of the following?

Dyskinesia

A client with a brain tumor is complaining of a headache upon awakening. Which nursing action would the nurse take first?

Elevate the head of the bed

A client is being admitted to a rehabilitation hospital as a result of the tetraplegia caused a stroke. The client's condition is stable, and after admission the client will begin physical and psychological therapy. An important part of nursing management is to reposition the client every 2 hours. What is the rationale behind this intervention?

maintain sufficient integument capillary pressure

A patient is treated for a neurologic dysfunction affecting facial expressions. The affected cranial nerve originates in the:

pons.

The nurse is instructing a community class when a student asks, "How does someone get super strength in an emergency?" The nurse should respond by describing the action of the:

sympathetic nervous system.

A client with a cerebellar brain tumor is admitted to an acute care facility. The nurse formulates a nursing diagnosis of Risk for injury. Which "related-to" phrase should the nurse add to complete the nursing diagnosis statement?

Related to impaired balance

The geriatric advanced practice nurse (APN) is doing client teaching with a client who has had a cerebrovascular accident (CVA) and the client's family. One concern the APN addresses is a potential for falls related to the CVA and resulting muscle weakness. What would be most important for the APN to include in teaching related to this concern?

Remove throw rugs and electrical cords from home environment.

A client is receiving an IV infusion of mannitol (Osmitrol) after undergoing intracranial surgery to remove a brain tumor. To determine whether this drug is producing its therapeutic effect, the nurse should consider which finding most significant?

Increased urine output

A 58-year-old construction worker fell from a 25-foot scaffolding and incurred a closed head injury as a result. As his intracranial pressure continues to increase, the potential of herniation also increases. If the brain herniates, which of the following are potential consequences? Choose all correct options.

Death Permanent neurologic dysfunction Impaired cellular activity

A client has undergone a lumbar puncture as part of a neurological assessment. The client is put under the care of a nurse after the procedure. Which important postprocedure nursing intervention should be performed to ensure the client's maximum comfort?

Encourage the client to drink liberal amounts of fluids

Which term is used to describe edema of the optic nerve?

Papilledema

A client with a malignant glioma is scheduled for surgery. The client demonstrates a need for additional teaching about the surgery when he states which of the following?

"The surgeon will be able to remove all of the tumor."

Lower motor neuron lesions cause a) no muscle atrophy. b) hyperactive and abnormal reflexes. c) flaccid muscle paralysis. d) increased muscle tone.

C, Lower motor neuron lesions cause flaccid muscle paralysis, muscle atrophy, decreased muscle tone, and loss of voluntary control. Upper motor neuron lesions cause increased muscle tone. Upper motor neuron lesions cause no muscle atrophy. Upper motor neuron lesions cause hyperactive and abnormal reflexes.

A client arrives at the emergency department complaining of extreme muscle weakness after minimal effort. The physician suspects myasthenia gravis. Which drug will be used to test for this disease? a) Carbachol (Carboptic) b) Edrophonium (Tensilon) c) Ambenonium (Mytelase) d) Pyridostigmine (Mestinon)

B, Edrophonium temporarily blocks the breakdown of acetylcholine, thus increasing acetylcholine level in the blood, and relieves weakness. Because of its short duration of action, edrophonium is the drug of choice for diagnosing myasthenia gravis. It's also used to differentiate myasthenia gravis from cholinergic toxicity. Ambenonium is used as an antimyasthenic. Pyridostigmine serves primarily as an adjunct in treating severe anticholinergic toxicity; it's also an antiglaucoma agent and a miotic. Carbachol reduces intraocular pressure during ophthalmologic procedures; topical carbachol is used to treat open-angle and closed-angle glaucoma.

A nurse is preparing a client for lumbar puncture. The client has heard about post-lumbar puncture headaches and asks how to avoid having one. The nurse tells the client that these headches can be avoided by doing which of the following after the procedure? a) "Remain NPO for 6 hours." b) "Ambulate as soon as possible." c) "Remain prone for 2 to 3 hours." d) "Remain on bedrest for 3 days."

C,The headache is caused by cerebral spinal fluid (CSF) leakage at the puncture site. The supply of CSF in the cranium is depleted so that there is not enough to cushion and stabilize the brain. When the client assumes an upright position, tension and stretching of the venous sinuses and pain-sensitive structures occur. The headache may be avoided if the client remains prone for 2 to 3 hours after the procedure. Drinking plenty of fluids will help in replacing the CSF. Hydration is important for replacement of the CSF lost so remaining NPO is not an option unless it is for other reasons, then IV fluid replacement will be important. Ambulating right away will make the possibility of a headache more likely. It is not necessary to remain on bedrest for more than a few hours, unless a headache has occurred; then bedest for overnight is usually sufficient

A 53-year-old man presents to the emergency department with a chief complaint of inability to form words, and numbness and weakness of the right arm and leg. Where would you locate the site of injury? a) Left basal ganglia b) Left frontoparietal region c) Left temporal region d) Right frontoparietal region

B, The patient is exhibiting signs of expressive aphasia with numbness/tingling and weakness of the right arm and leg. This indicates injury to the expressive speech center (Broca's area), which is located in the inferior portion of the frontal lobe. The motor strip is located in the posterior portion of the frontal lobe. The sensory strip is located in the anterior parietal lobe.

A client newly diagnosed with multiple sclerosis (MS) asks about a cure for her disease process. The nurse gives which of the following information? a) Medications do not assist with relief of signs and symptoms. b) If recommendations for symptom relief are followed, the disease will be cured. c) There is no cure for MS. d) Life expectancy for clients with MS is dramatically different from that of those without MS.

C, No cure exists for MS. Life expectancy for clients with MS is not dramatically different from that of clients without MS. Medications are available for symptom management of clients with MS

A nurse is planning discharge for a client who experienced right-sided weakness caused by a stroke. During his hospitalization, the client has been receiving physical therapy, occupational therapy, and speech therapy daily. The family voices concern about rehabilitation after discharge. How should the nurse intervene?

Inform the case manager of the family's concern and provide information about the client's current clinical status so appropriate resources can be provided after discharge.

The nurse in the emergency department is caring for a patient brought in by the rescue squad after falling from a second-story window. The nurse assesses ecchymosis over the mastoid and clear fluid from the ears. What type of skull fracture is this indicative of?

Basilar skull fracture

A patient with Bell's palsy says to the nurse, "It doesn't hurt anymore to touch my face. How am I going to get muscle tone back so I don't look like this anymore?" What interventions can the nurse suggest to the patient? a) Tell the patient to smile every 4 hours. b) Suggest massaging the face several times daily, using a gentle upward motion, to maintain muscle tone. c) Inform the patient that the muscle function will return as soon as the virus dissipates. d) Suggest applying cool compresses on the face several times a day to tighten the muscles.

B, After the sensitivity of the nerve to touch decreases and the patient can tolerate touching the face, the nurse can suggest massaging the face several times daily, using a gentle upward motion, to maintain muscle tone. Facial exercises, such as wrinkling the forehead, blowing out the cheeks, and whistling, may be performed with the aid of a mirror to prevent muscle atrophy. Exposure of the face to cold and drafts is avoided

The nurse is caring for a patient who was involved in a motor vehicle accident and sustained a head injury. When assessing deep tendon reflexes (DTR), the nurse observes diminished or hypoactive reflexes. How will the nurse document this finding?

1+

A client with newly diagnosed seizures asks about stigma associated with epilepsy. The nurse will respond with which of the following statements?

"Many people with developmental disabilities resulting from neurologic damage also have epilepsy."

A nurse is preparing a client for lumbar puncture. The client has heard about post-lumbar puncture headaches and asks how to avoid having one. The nurse tells the client that these headches can be avoided by doing which of the following after the procedure?

"Remain prone for 2 to 3 hours."

A patient is admitted via ambulance to the emergency room of a stroke center at 1:30 p.m. with symptoms that the patient said began at 1:00 p.m. Within 1 hour, an ischemic stroke had been confirmed and the doctor ordered tPA. The nurse knows to give this drug no later than what time?

4:00 pm

A client with cerebral metastasis suddenly experiences a seizure for which phenytoin 10 mg/kg intravenously is ordered as an initial loading dose. The client weighs 132 pounds. How many milligrams of phenytoin should the client receive? Enter the number ONLY.

600

A client is receiving intravenous (IV) mannitol to prevent increased intracranial pressure. The order is for mannitol 1.5 grams per kg of body weight IV now. The client weighs 143 lbs (65 kg). How many grams will the nurse administer to the client? Enter the correct number ONLY.

97.5

You are a neurotrauma nurse working in a neuro ICU. What would you know is an acute emergency and is seen in clients with a cervical or high thoracic spinal cord injury after the spinal shock subsides?

Autonomic dysreflexia

One defining characteristic of a complex partial seizure versus a simple partial seizure is the presence of which of the following? a) Impaired consciousness b) Motor symptoms c) Sensory symptoms d) Compound forms

A, A complex partial seizure is characterized by complex symptoms with the impairment of consciousness. A simple partial seizure generally occurs without impairment of consciousness.

The nurse is caring for a patient with MS who is having spasticity in the lower extremities that decreases physical mobility. What interventions can the nurse provide to assist with relieving the spasms? (Select all that apply.) a) Allow the patient adequate time to perform exercises b) Demonstrate daily muscle stretching exercises. c) Assist with a rigorous exercise program to prevent contractures. d) Have the patient take a hot tub bath to allow muscle relaxation. e) Apply warm compresses to the affected areas.

A,B,E Warm packs may be beneficial for relieving spasms, but hot baths should be avoided because of risk of burn injury secondary to sensory loss and increasing symptoms that may occur with elevation of the body temperature. Daily exercises for muscle stretching are prescribed to minimize joint contractures. The patient should not be hurried in any of these activities, because this often increases spasticity.

A client is scheduled for an EEG. The client asks about any diet-related prerequisites before the EEG. Which diet-related advice should the nurse provide to the client?

Avoid taking sedative drugs or drinks that contain caffeine for at least 8 hours before the test.

A nurse is teaching a client with multiple sclerosis (MS). When teaching the client how to reduce fatigue, the nurse should tell the client to: a) rest in an air-conditioned room. b) avoid naps during the day. c) take a hot bath. d) increase the dose of muscle relaxants.

A,Fatigue is a common symptom in clients with MS. Lowering the body temperature by resting in an air-conditioned room may relieve fatigue; however, extreme cold should be avoided. A hot bath or shower can increase body temperature, producing fatigue. Muscle relaxants, ordered to reduce spasticity, can cause drowsiness and fatigue. Frequent rest periods and naps can relieve fatigue. Other measures to reduce fatigue in the client with MS include treating depression, using occupational therapy to learn energy-conservation techniques, and reducing spasticity.

A nurse is caring for a client with a brain tumor and increased intracranial pressure (ICP). Which intervention should the nurse include in the care plan to reduce ICP?

Administer stool softeners

Which term refers to the inability to recognize objects through a particular sensory system?

Agnosia

A client undergoes a scheduled electroencephalogram (EEG). Which of the following post-procedure activities should the nurse carry out for the client?

Allow the client to rest and shampoo the client's hair.

A client suffers a head injury. The nurse implements an assessment plan to monitor for potential subdural hematoma development. Which manifestation does the nurse anticipate seeing first?

Alteration in level of consciousness (LOC)

Which disease includes loss of motor neurons in the anterior horns of the spinal cord and motor nuclei of the lower brain stem?

Amyotrophic lateral sclerosis

The nurse reviews the physician's emergency department progress notes for the client who sustained a head injury and sees that the physician observed the Battle sign. The nurse knows that the physician observed which clinical manifestation?

An area of bruising over the mastoid bone

A patient comes to the emergency department with severe pain in the face that was stimulated by brushing the teeth. What cranial nerve does the nurse understand can cause this type of pain? a) VI b) V c) III d) IV

B, The trigeminal nerve (cranial nerve V) innervates the forehead, cheeks, and jaw, so pain in the face elicited when brushing the teeth would most likely involve this nerve.

The sympathetic and parasympathetic nervous systems have a direct affect on the circulatory system. Stimulation of the parasympathetic nervous system (PNS) causes which of the following? a) Blood vessels in the heart muscle to dilate b) Heartbeat to decrease c) Blood pressure to increase d) Blood vessels in the skeletal muscles to dilate

B,The parasympathetic nervous system has a constricting effect on the blood vessels in the heart and skeletal muscles; the heartbeat and blood pressure will decrease

The nurse is caring for a client who was diagnosed with a glioma 5 months ago. Today, the client was brought to the emergency department by his caregiver because he collapsed at home. The nurse suspects late signs of rising intracranial pressure (ICP) when which blood pressure and pulse readings are noted?

BP = 175/45 mm Hg; HR = 42 bpm

At a certain point, the brain's ability to autoregulate becomes ineffective and decompensation (ischemia and infarction) begins. Which of the following are associated with Cushing's triad? Select all that apply.

Bradycardia Hypertension Bradypnea

The nurse is caring for a patient in the emergency department with a diagnosed epidural hematoma. What procedure will the nurse prepare the patient for?

Burr holes

A patient diagnosed with MS 2 years ago has been admitted to the hospital with another relapse. The previous relapse was followed by a complete recovery with the exception of occasional vertigo. What type of MS does the nurse recognize this patient most likely has? a) Disabling b) Primary progressive c) Relapsing-remitting (RR) d) Benign

C, Approximately 85% of patients with MS have a relapsing-remitting (RR) course. With each relapse, recovery is usually complete; however, residual deficits may occur and accumulate over time, contributing to functional decline.

The nurse is performing an assessment of cranial nerve function and asks the patient to cover one nostril at a time to see if the patient can smell coffee, alcohol, and mint. The patient is unable to smell any of the odors. The nurse is aware that the patient has a dysfunction of which cranial nerve?

CN I

The nurse obtains a Snellen eye chart when assessing cranial nerve function. Which cranial nerve is the nurse testing when using the chart?

CN II

A patient presents to the emergency room with complaints of having an "exploding headache" for the last 2 hours. The patient is immediately seen by a triage nurse who suspects the patient is experiencing a stroke. Which of the following is a possible cause based on the characteristic symptom?

Cerebral aneurysm

The physician's office nurse is caring for a client who has a history of a cerebral aneurysm. Which diagnostic test does the nurse anticipate to monitor the status of the aneurysm?

Cerebral angiography

When performing a postoperative assessment on a client who has undergone surgery to manage increased intracranial pressure (ICP), a nurse notes an ICP reading of 0 mm Hg. Which action should the nurse perform first?

Check the equipment.

A nurse is caring for a patient with a diagnosis of trigeminal neuralgia. Which activity is altered as a result of this diagnosis?

Chewing

A patient has been diagnosed with a lipoma. The nurse explains to the patient that this tumor is located in the part of the brain known as the:

Corpus callosum.

During a Tensilon test to determine if a patient has myasthenia gravis, the patient complains of cramping and becomes diaphoretic. Vital signs are BP 130/78, HR 42, and respiration 18. What intervention should the nurse prepare to do? a) Place the patient in the supine position. b) Administer diphenhydramine (Benadryl) for the allergic reaction. c) Call the rapid response team because the patient is preparing to arrest. d) Administer atropine to control the side effects of edrophonium.

D, Atropine should be available to control the side effects of edrophonium, which include bradycardia, sweating, and cramping.

A client with a suspected brain tumor is scheduled for a computed tomography (CT) scan. What should the nurse do when preparing the client for this test?

Determine whether the client is allergic to iodine, contrast dyes, or shellfish.

A nurse is caring for a client with deteriorating neurologic status. The nurse is performing an assessment at the beginning of the shift that reveals a falling blood pressure and heart rate, and the client makes no motor response to stimuli. Which documentation of neuromuscular status is most appropriate?

Flaccidity

Which cerebral lobes is the largest and controls abstract thought?

Frontal

A client is experiencing muscle weakness and an ataxic gait. The client has a diagnosis of multiple sclerosis (MS). Based on these symptoms, the nurse formulates "Impaired physical mobility" as one of the nursing diagnoses applicable to the client. What nursing intervention should be most appropriate to address the nursing diagnosis?

Help the client perform range-of-motion (ROM) exercises every 8 hours.

Which is a contraindication for the administration of tissue plasminogen activator (t-PA)?

Intracranial hemorrhage

Which are contraindications for the administration of tissue plasminogen activator (t-PA)? Select all that apply.

Intracranial hemorrhage Major abdominal surgery within 10 days

A client has experienced an ischemic stroke that has damaged the lower motor neurons of the brain. Which of the following deficits would the nurse expect during assessment?

Lack of deep tendon reflexes

A client has experienced an ischemic stroke that has damaged the frontal lobe of his brain. Which of the following deficits does the nurse expect to observe during assessment?

Limited attention span and forgetfulness

A client was undergoing conservative treatment for a herniated nucleus pulposus, at L5 - S1, which was diagnosed by magnetic resonance imaging. Because of increasing neurologic symptoms, the client undergoes lumbar laminectomy. The nurse should take which step during the immediate postoperative period?

Logroll the client from side to side.

The nurse is caring for a patient immediately following supratentorial intracranial surgery. What action by the nurse is appropriate?

Place patient in supine position with head slightly elevated.

The nurse is caring for a client diagnosed with a subarachnoid hemorrhage resulting from a leaking aneurysm. The client is awaiting surgery. Which nursing interventions would be appropriate for the nurse to implement? Select all that apply.

Provide a dimly lit environment. Elevate the head of bed 30 degrees. Administer docusate per order.

The nurse is providing diet-related advice to a male patient following a cerebrovascular accident (CVA). The patient wants to minimize the volume of food and yet meet all nutritional elements. Which of the following suggestions should the nurse give to the patient about controlling the volume of food intake?

Provide thickened commercial beverages and fortified cooked cereals.

The nurse is caring for a patient with Huntington's disease in the long-term care facility. What does the nurse recognize as the most prominent symptom of the disease that the patient exhibits?

Rapid, jerky, involuntary movements

A nurse completes the Glasgow Coma Scale on a patient with traumatic brain injury (TBI). Her assessment results in a score of 6, which is interpreted as:

Severe TBI.

The nurse is caring for a client with a significant allergy history to various medications and shellfish. Because the client needs to have a diagnostic study with contrast, which medication classification is anticipated?

antihistamine

The nurse practitioner advises a patient who is at high risk for a stroke to be vigilant in his medication regimen, to maintain a healthy weight, and to adopt a reasonable exercise program. This advice is based on research data that shows the most important risk factor for stroke is:

hypertension

A client admitted to the emergency department is being evaluated for the possibility of a stroke. Which assessment finding would lead the nurse to suspect that the client is experiencing a hemorrhagic stroke?

severe exploding headache

The nurse is participating in a health fair for stroke prevention. Which will the nurse say is a modifiable risk factor for ischemic stroke?

smoking

A nurse in a rehabilitation facility is coordinating the discharge of a client who is tetraplegic. The client, who is married and has two children in high school, is being discharged to home and will require much assistance. Who would the discharge planner recognize as being the most important member of this client's care team?

spouse

During a routine physical examination to assess a client's deep tendon reflexes, a nurse should make sure to:

support the joint where the tendon is being tested.

The nurse is caring for a client hospitalized after a motor vehicle accident. The client has a comorbidity of Parkinson's disease. Why should the nurse closely monitor the condition and the drug regimen of a client with Parkinson's disease?

Drugs administered may cause a wide variety of adverse effects.

The nurse is taking care of a patient with a history of headaches. The nurse takes measures to reduce headaches in the patient in addition to administering medications. Which of the following appropriate nursing interventions may be provided by the nurse to such a patient?

Apply warm or cool cloths to the forehead or back of the neck

A client has been admitted for observation after a closed head injury. There is clear fluid leaking from the client's nose. How would the nurse assess if this drainage is CSF?

Assess for a halo sign

A client has experienced an ischemic stroke that has damaged the temporal (lateral and superior portions) lobe. Which of the following deficits would the nurse expect during assessment of this client?

Auditory agnosia

Which of the following positions are employed to help reduce intracranial pressure (ICP)?

Avoiding flexion of the neck with use of a cervical collar

What does the nurse recognize as the earliest sign of serious impairment of brain circulation related to increasing ICP? a) Bradycardia b) Lethargy and stupor c) A bounding pulse d) Hypertension

B, As ICP increases, the patient becomes stuporous, reacting only to loud or painful stimuli. At this stage, serious impairment of brain circulation is probably taking place, and immediate intervention is required

The nurse is educating a patient with a seizure disorder. What nutritional approach for seizure management would be beneficial for this patient? a) Low in fat b) High in protein and low in carbohydrate c) Restricts protein to 10% of daily caloric intake d) At least 50% carbohydrate

B, A dietary intervention, referred to as the ketogenic diet, may be helpful for control of seizures in some patients. This high-protein, low-carbohydrate, high-fat diet is most effective in children whose seizures have not been controlled with two antiseizure medications, but it is sometimes used for adults who have had poor seizure control (Mosek, Natour, Neufeld, et al., 2009).

The nurse is seeing a female client who has been diagnosed with a pituitary adenoma. During the clinic visit, the client tells the nurse that she has been having irregular menstrual periods despite having very regular menstrual periods all her life. The nurse knows this physiological change is likely related to which characteristic of this type of brain tumor?

Increased prolactin levels

A nurse is monitoring a client for increasing intracranial pressure (ICP). Early signs of increased ICP include: a) decreasing blood pressure. b) elevated temperature. c) pupillary changes. d) diminished responsiveness.

D, Usually, diminished responsiveness is the first sign of increasing ICP. Pupillary changes occur later. Increased ICP causes systolic blood pressure to rise. Temperature changes vary and may not occur even with a severe decrease in responsiveness.

A patient 3 days postoperative from a craniotomy informs the nurse, "I feel something trickling down the back of my throat and I taste something salty." What priority intervention does the nurse initiate? a) Give the patient some mouthwash to gargle with. b) Request an antihistamine for the postnasal drip. c) Ask the patient to cough to observe the sputum color and consistency. d) Notify the physician of a possible cerebrospinal fluid leak.

D,Any sudden discharge of fluid from a cranial incision is reported at once, because a large leak requires surgical repair. Attention should be paid to the patient who complains of a salty taste or "postnasal drip," because this can be caused by cerebrospinal fluid trickling down the throat.

Working hard to memorize the functions of the cranial nerves is a typical part of nursing school. Not only is it important to correlate the proper nerve number and name, but including the proper function makes this task quite a challenge! Which cranial nerves are enabling you to read this question? a) Oculomotor b) Abducens c) Trochlear d) All options are correct

D,The Oculomotor (III), Abducens (VI) and Trochlear (IV) nerves all work within the functional realm of the eyes. Don't forget the Optic (II) nerve!

After the patient has received tPA, the nurse knows to check vital signs every 30 minutes for 6 hours. Which of the following readings would require calling the provider?

Diastolic pressure of 110 mm Hg

The nurse is caring for a client with a history of transient ischemic attacks (TIAs) and moderate carotid stenosis who has undergone a carotid endarterectomy. Which postoperative finding would cause the nurse the most concern?

Difficulty swallowing

A patient is exhibiting bradykinesia, rigidity, and tremors related to Parkinson's disease. The nurse understands that these symptoms are directly related to what decreased neurotransmitter level?

Dopamine

A nurse is reviewing a CT scan of the brain, which states that the client has arterial bleeding with blood accumulation above the dura. Which of the following facts of the disease progression is essential to guide the nursing management of client care?

Monitoring is needed as rapid neurologic deterioration may occur.

A client diagnosed with a stroke is ordered to receive warfarin. Later, the nurse learns that the warfarin is contraindicated and the order is canceled. The nurse knows that the best alternative medication to give is

aspirin

When should the nurse plan the rehabilitation of a patient who is having an ischemic stroke?

the day the patient has the stroke

A client is actively hallucinating during an assessment. The nurse would be correct in documenting the hallucination as a disturbance in

thought content.

The brain stem holds the medulla oblongata. What is the function of the medulla oblongata?

transmits motor impulses from the brain to the spinal cord

A client has been diagnosed with a concussion and is to be released from the emergency department. The nurse teaches the family or friends who will be caring for the client to contact the physician or return to the ED if the client

vomits

Which term refers to the inability to perform previously learned purposeful motor acts on a voluntary basis?

Apraxia

Impaired balance and uncontrolled tremors of Parkinson's disease is correlated with which neurotransmitter?

Dopamine

A patient is being tested for a gag reflex. When the nurse places the tongue blade to the back of the throat, there is no response elicited. What dysfunction does the nurse determine the patient has?

Dysfunction of the vagus nerve

A client is experiencing dysphagia following a stroke. Which measure may be taken by the nurse to ensure that the client's diet allows for easy swallowing?

Help the client sit upright when eating and feed slowly.

The nurse is educating a patient with a seizure disorder. What nutritional approach for seizure management would be beneficial for this patient?

High in protein and low in carbohydrate

A nurse is providing education to a community group about ischemic strokes. One group member asks if there are ways to reduce the risk for stroke. Which of the following is a risk factor that can be modified?

Hypertension

Bone density testing will be completed for the client with post-polio syndrome. The nurse teaches the client bone density testing is used to identify what potential complication?

Low bone mass and osteoporosis

A client with spinal cord compression from a tumor must undergo diagnostic testing. Which of the following is the most likely procedure for this client?

Magnetic resonance imaging

Which diagnostic is most commonly used for spinal cord compression?

Magnetic resonance imaging (MRI)

Which of the following is accurate regarding a hemorrhagic stroke?

Main presenting symptom is an "exploding headache."

A patient is brought to the emergency department with a possible stroke. What initial diagnostic test for a stroke, usually performed in the emergency department, would the nurse prepare the patient for?

Noncontrast computed tomogram

When educating a patient about the use of antiseizure medication, what should the nurse inform the patient is a result of long-term use of the medication in women?

Osteoporosis

A patient is admitted to the hospital for management of an extrapyramidal disorder. Included in the physician's admitting orders are the medications levodopa, Cogentin, and Eldepryl. The nurse knows that most likely, the client has a diagnosis of ________.

Parkinson's disease


Conjuntos de estudio relacionados

Topic 12.3 Financing Government with Taxes

View Set

Ins. Exam: Field Underwriting Procedures

View Set

Chapter 15 ( Bone health and body defense systems)

View Set

Chapter 9 Physical Science (What Is Radioactivity)

View Set

Chapter 2: WORKFORCE SAFETY AND WELLNESS

View Set

Prep u fluid and electrolytes (exam 2 nurs 301)

View Set

Health Chapter 4 Section 2 Review and Vocab

View Set